The Constant Horizontal Force Needed to Raise the Wheel

  • Thread starter e(ho0n3
  • Start date
  • Tags
    Wheel
In summary, the minimum horizontal force needed to make a wheel of mass M and radius R climb a step of height h (where h < R) is F = mg \frac{\sqrt{2hR - h^2}}{R-h}. However, this only applies to the initial force needed and the force required will vary as the wheel rolls up the step. The original analysis using energy methods was not accurate, as it assumed a constant force and neglected the wheel's kinetic energy.
  • #1
e(ho0n3
1,357
0
Just need to know if I'm on the right track (again):

A wheel of mass M has radius R. It is standing vertically on the floor, and we want to exert a horizontal force F at its axle so that it will climb a step against which it rests. The step has height h, where h < R. What minimum force F is needed?

This problem was whispering to me: "Use energy methods...". The wheel moves about the corner of the step so I could take that as my axis of rotation and calculate the torque about it. The I proceeded to calculate the work done by this torque, i.e. [itex] W = \int{\tau} d\theta[/itex], to get the wheel up the step and equate this with the negative change of potential energy (since [itex]-W = \Delta{U}[/itex]) of the center of mass so that I could solve for F. This gave me the following (assuming I didn't screw up somewhere):
[tex]
F = \frac{mgh(1-R)}{\sqrt{2hR - h^2}}
[/tex]​
For F to be 'valid' I must have [itex]2hR - h^2 > 0 \Rightarrow R > h/2[/itex]. This is what worries me since I was explicitly given that R > h. I know what I did is wrong then, but I don't know what to do now. Any tips?
 
Physics news on Phys.org
  • #2
Torque!

Consider that the wheel pivots about the corner of the step. So take moments about that point. The horizontal force F must exert enough torque to overcome the torque exerted by the wheel's weight.
 
  • #3
Your method looks fine to me, so I'm assuming that you probably did make an error in the algebra. I remember having this problem on a final a couple quarters back and the best advice I could give would be to draw a good picture, with all the angles in place so you can see the relationships clearly. And just a little tip; as soon as problems begin whispering to you, please take a break, because you're studying too hard. :smile:
 
  • #4
Doc Al said:
Consider that the wheel pivots about the corner of the step. So take moments about that point. The horizontal force F must exert enough torque to overcome the torque exerted by the wheel's weight.
Let [itex]\tau_F[/itex] and [itex]\tau_w[/itex] be the torques produced by F and the weight respectively. My understanding of your response is telling me that [itex]\tau_F > \tau_w[/itex], but then this doesn't make sense so maybe I don't understand (nothing new there).

I did this problem again by setting [itex]\tau_F - \tau_w = I\alpha[/itex] and then figuring out what [itex]\alpha[/itex] is using [itex]\alpha = a/R[/itex] and of course I can find [itex]a[/itex] using Netwon's 2. law. This gives me
[tex]
F = \frac{\sqrt{2hR - h^2}}{R-h}
[/tex]​
but of course this implies that [itex]R \geq h/2[/itex].
 
Last edited:
  • #5
e(ho0n3 said:
Let [itex]\tau_F[/itex] and [itex]\tau_w[/itex] be the torques produced by F and the weight respectively. My understanding of your response is telling me that [itex]\tau_F > \tau_w[/itex], but then this doesn't make sense so maybe I don't understand (nothing new there).
What doesn't make sense? The minimum horizontal force needed will be just enough so that [itex]\tau_F = \tau_w[/itex]
I did this problem again by setting [itex]\tau_F - \tau_w = I\alpha[/itex] and then figuring out what [itex]\alpha[/itex] is using [itex]\alpha = a/R[/itex] and of course I can find [itex]a[/itex] using Netwon's 2. law. This gives me
[tex]
F = \frac{\sqrt{2hR - h^2}}{R-h}
[/tex]​
but of course this implies that [itex]R \geq h/2[/itex].
No need to calculate α; just set the torques equal. You left out the weight of the wheel:
[tex]
F = mg \frac{\sqrt{2hR - h^2}}{R-h}
[/tex]​
And don't forget that the denominator implies [itex]R > h[/itex]
 
Last edited:
  • #6
Doc Al said:
What doesn't make sense? The minimum horizontal force needed will be just enough so that [itex]\tau_F = \tau_w[/itex]
Come to think of it, I don't remember why I said that. If the wheel rotates about the corner of the step, then the angular acceleartion is greater than 0 so [itex]\tau_F - \tau_w > 0[/itex].
No need to calculate α; just set the torques equal. You left out the weight of the wheel:
[tex]
F = mg \frac{\sqrt{2hR - h^2}}{R-h}
[/tex]​
And don't forget that the denominator implies [itex]R > h[/itex]
I've scribbled so much that the mg term 'magically' disappeared. I trust this is the answer then. But why did my original analysis not yield the same answer? I think everything I did was legal wasn't it?
 
  • #7
e(ho0n3 said:
But why did my original analysis not yield the same answer? I think everything I did was legal wasn't it?
Since the minimum force is not constant as the wheel rolls up the step, it's not obvious to me how to apply energy methods to this problem in a simple manner. We only need to find the initial force to get the wheel up the step.
 
  • #8
e(ho0n3 said:
But why did my original analysis not yield the same answer? I think everything I did was legal wasn't it?

No, unfortunately, everything was not legal. For instance, if you are assuming a constant force F, then the wheel WILL have a KE after it has climbed the step. So it's not right to say that all the work done went into raising the PE.

Also, there's clearly some algebraic error in your original approach since the answer is dimensionally wonky ! You cannot have a (1-R) term - what dimension does that have ?
 
  • #9
Gokul43201 said:
No, unfortunately, everything was not legal. For instance, if you are assuming a constant force F, then the wheel WILL have a KE after it has climbed the step. So it's not right to say that all the work done went into raising the PE.
Exactly right. The horizontal force needed to just raise the wheel with no added KE is not constant. (That was the point I was trying to make in my last post.)
 

1. How does a wheel climb a step?

A wheel climbs a step by using its circular shape and rotation to distribute its weight and create momentum to move up the step. The wheel's contact with the ground also provides friction, which helps it to grip and move up the step.

2. What factors affect a wheel's ability to climb a step?

The size and shape of the wheel, the height and angle of the step, and the material and surface of the wheel and step are all factors that can affect a wheel's ability to climb a step. Additionally, the weight and distribution of weight on the wheel can also play a role.

3. Can any type of wheel climb a step?

No, not all types of wheels are designed to climb steps. Wheels with a smaller diameter and larger width are typically better suited for climbing steps, as they have a larger contact area with the ground and can better distribute their weight.

4. Are there any real-world applications for wheel climbing a step?

Yes, wheel climbing a step is commonly used in robotics and engineering, where it can be applied to create more efficient and versatile machines. It is also seen in everyday objects such as strollers and carts, where the ability to climb steps makes them more convenient for use in different environments.

5. Can a wheel climb multiple steps at once?

It is possible for a wheel to climb multiple steps at once, but it depends on the size and shape of the wheel, the height and angle of the steps, and the weight and distribution of weight on the wheel. In most cases, a wheel will have an easier time climbing one step at a time.

Similar threads

  • Introductory Physics Homework Help
Replies
1
Views
480
  • Introductory Physics Homework Help
Replies
4
Views
592
  • Introductory Physics Homework Help
Replies
7
Views
2K
  • Introductory Physics Homework Help
Replies
6
Views
156
  • Introductory Physics Homework Help
Replies
16
Views
1K
  • Introductory Physics Homework Help
Replies
6
Views
3K
  • Introductory Physics Homework Help
Replies
8
Views
1K
Replies
12
Views
1K
  • Introductory Physics Homework Help
Replies
32
Views
1K
  • Introductory Physics Homework Help
Replies
2
Views
861
Back
Top